Answer to an SHM Problem Does Not Fit Into Original Situation

  • Thread starter modulus
  • Start date
  • Tags
    Fit Shm
In summary, the amplitude of a particle in SHM remains constant, but the kinetic energy increases by an amount (0.5)mω2A2. The new amplitude becomes equal to (√5)Aω.
  • #1
modulus
127
3

Homework Statement


A particle performs SHM of amplitude A along a straight line. When it is at a distance (√3/2)A from the mean position, its kinetic energy gets increased by an amount (0.5)mω2A2 due to an impulsive force. Then its new amplitude becomes?


Homework Equations


The basic idea is that the total energy (KE + PE) of a system in SHM remains constant.

Also of use:
v = Aω*cos(ωt)
vmax = Aω, in an SHM of angular frequency ω and amplitude A.
Kinetic Energy = 1/2(mv2)

The Attempt at a Solution


So, once the system gains the given amount of extra energy, it's total energy becomes mω2A'2.
Which means the amplitude A' will be equal to (√2)A (that is assuming that the value of ω does not change...which makes complete sense).

But, I run into a little problem when I try to use this result in the original situation:
I figured out that the speed of the particle at the position given (before the impulse) would be Aω/2. After the addition of kinetic energy given in the problem, the speed will become [itex]\frac{(√5)Aω}{2}[/itex]. Comparing this with the first equation I've listed, that implies that A' = (√5)A (because ω doesn't change).

...What went wrong??
 
Physics news on Phys.org
  • #2
Prior to the impulse, the physical position of the mass corresponded to some angle ωt. That is,

## \frac{\sqrt{3}}{2}A = A sin(ω t)##

and angle θ = ωt turned out to be 60 degrees.

After the impulse adds energy to the system, the amplitude increases and this particular physical location no longer corresponds to an angle of 60 degrees in the cycle, so you can't assume that cos(ωt) will still be 1/2.

Edit: In order to maintain timeline continuity, the "new" equation for the position of the object would have to incorporate a phase angle so that the 'before' and 'after' equations meet seamlessly at the impulse instant. Suppose the impulse occurs at time tx, then

##x(t) = Asin(ω t) ~~~~~~~~~~~~~~0 \le t < t_x##
##x(t) = A'sin(ω t + \phi)~~~~~~~~~~t \ge t_x##
 
Last edited:
  • #3
The kinetic energy at that point is 1/8 m ω^2 A^2. You increase it by 1/2 m ω^2A^2, so you increase its velocity by a factor of sqrt(5).

The position where the particle gets its kick is ##\frac{\sqrt{3}{2}}## of the initial amplitude, this is ##\sqrt\frac{3}{8}}## of the new amplitude (using A'=√2*A). There, the particle has 5/8 of its total energy as kinetic energy and its velocity is the value you calculated.

Short version:
(√3/2)A != (√3/2)A'
 
  • #4
OK, so this is where I was going wrong:


A'ω cos(new phase angle) = ((√5)/2)Aω

And the "new phase angle" will adjust itself so that this relation is true for A' = A√2.
In fact, the "new phase angle" will be such that:

A' sin(new phase angle) = ((√3)/2)A


Oh...clarity!
Thanks, guys. :)
 
  • #5


I would first check if all the given information is accurate and if there are any discrepancies. I would also recheck my calculations and equations to make sure they are correct. If I still cannot find a solution, I would consider the possibility that the problem may have been incorrectly stated or that there is missing information. In this case, it seems like there may be a discrepancy between the given amount of increased kinetic energy and the resulting change in amplitude. It is also possible that the problem assumes certain conditions or simplifications that are not explicitly stated. To fully understand and solve this problem, I would need more information and clarification.
 

1. What is SHM and why is it important?

SHM stands for Simple Harmonic Motion, which is a type of periodic motion where an object oscillates back and forth around a fixed equilibrium point. It is important because many natural phenomena, such as vibrations, waves, and orbits, can be described as SHM and understanding it helps us understand the world around us.

2. What does it mean when the answer to an SHM problem does not fit into the original situation?

This means that the answer to the SHM problem does not make sense in the context of the original situation. It could be due to incorrect assumptions, incorrect equations used, or other factors. It is important to carefully check the problem and solution to identify the source of the discrepancy.

3. How can I ensure that my answer to an SHM problem fits into the original situation?

To ensure that your answer fits into the original situation, it is important to carefully read and understand the problem, identify the correct equations to use, and double-check your calculations. You can also try plugging your answer back into the original equation to see if it makes sense.

4. What are some common mistakes that can lead to an answer not fitting into the original situation in SHM problems?

Some common mistakes include using incorrect equations, making incorrect assumptions about the problem, and using incorrect values for variables. It is also important to pay attention to units and ensure they are consistent throughout the problem and solution.

5. What should I do if my answer does not fit into the original situation in an SHM problem?

If your answer does not fit into the original situation, carefully review your work and check for any mistakes. You can also try approaching the problem from a different angle or consulting with a classmate or teacher for help. It is important to understand the source of the discrepancy in order to learn from the mistake and improve in the future.

Similar threads

Replies
2
Views
653
  • Introductory Physics Homework Help
Replies
21
Views
2K
  • Introductory Physics Homework Help
Replies
1
Views
1K
  • Introductory Physics Homework Help
Replies
14
Views
1K
  • Introductory Physics Homework Help
Replies
10
Views
5K
  • Introductory Physics Homework Help
Replies
3
Views
1K
Replies
1
Views
578
Replies
10
Views
431
  • Introductory Physics Homework Help
Replies
5
Views
12K
  • Introductory Physics Homework Help
Replies
21
Views
1K
Back
Top